$$\newcommand{\mtn}{\mathbb{N}}\newcommand{\mtns}{\mathbb{N}^*}\newcommand{\mtz}{\mathbb{Z}}\newcommand{\mtr}{\mathbb{R}}\newcommand{\mtk}{\mathbb{K}}\newcommand{\mtq}{\mathbb{Q}}\newcommand{\mtc}{\mathbb{C}}\newcommand{\mch}{\mathcal{H}}\newcommand{\mcp}{\mathcal{P}}\newcommand{\mcb}{\mathcal{B}}\newcommand{\mcl}{\mathcal{L}} \newcommand{\mcm}{\mathcal{M}}\newcommand{\mcc}{\mathcal{C}} \newcommand{\mcmn}{\mathcal{M}}\newcommand{\mcmnr}{\mathcal{M}_n(\mtr)} \newcommand{\mcmnk}{\mathcal{M}_n(\mtk)}\newcommand{\mcsn}{\mathcal{S}_n} \newcommand{\mcs}{\mathcal{S}}\newcommand{\mcd}{\mathcal{D}} \newcommand{\mcsns}{\mathcal{S}_n^{++}}\newcommand{\glnk}{GL_n(\mtk)} \newcommand{\mnr}{\mathcal{M}_n(\mtr)}\DeclareMathOperator{\ch}{ch} \DeclareMathOperator{\sh}{sh}\DeclareMathOperator{\th}{th} \DeclareMathOperator{\vect}{vect}\DeclareMathOperator{\card}{card} \DeclareMathOperator{\comat}{comat}\DeclareMathOperator{\imv}{Im} \DeclareMathOperator{\rang}{rg}\DeclareMathOperator{\Fr}{Fr} \DeclareMathOperator{\diam}{diam}\DeclareMathOperator{\supp}{supp} \newcommand{\veps}{\varepsilon}\newcommand{\mcu}{\mathcal{U}} \newcommand{\mcun}{\mcu_n}\newcommand{\dis}{\displaystyle} \newcommand{\croouv}{[\![}\newcommand{\crofer}{]\!]} \newcommand{\rab}{\mathcal{R}(a,b)}\newcommand{\pss}[2]{\langle #1,#2\rangle} $$
Bibm@th

Exercices corrigés - Intégrales impropres - fonctions intégrables

Convergence
Exercice 1 - Convergence d'intégrales impropres - 1 [Signaler une erreur] [Ajouter à ma feuille d'exos]
Enoncé
Les intégrales impropres suivantes sont-elles convergentes? $$\begin{array}{lll} \displaystyle \mathbf 1.\ \int_0^1 \ln tdt&&\displaystyle \mathbf 2.\ \int_0^{+\infty}e^{-t^2}dt\\ \displaystyle \mathbf 3.\ \int_0^{+\infty}x(\sin x)e^{-x}dx&&\displaystyle \mathbf 4.\ \int_0^{+\infty}(\ln t)e^{-t}dt\\ \displaystyle \mathbf 5.\ \int_0^1 \frac{dt}{(1-t)\sqrt t} \end{array} $$
Indication
Corrigé
Exercice 2 - Convergence d'intégrales impropres - 2 [Signaler une erreur] [Ajouter à ma feuille d'exos]
Enoncé
Les intégrales impropres suivantes sont-elles convergentes? $$\begin{array}{lll} \displaystyle \mathbf 1.\ \int_0^{+\infty}\frac{dt}{e^t-1}&&\displaystyle \mathbf 2.\ \int_0^{+\infty}\frac{te^{-\sqrt t}}{1+t^2}dt\\ \displaystyle \mathbf 3. \int_0^1 \cos^2\left(\frac1t\right)dt \end{array}$$
Indication
Corrigé
Exercice 3 - Convergence d'intégrales impropres - 3 [Signaler une erreur] [Ajouter à ma feuille d'exos]
Enoncé
Les intégrales impropres suivantes sont-elles convergentes? $$\begin{array}{lll} \displaystyle \mathbf 1.\ \int_0^{+\infty}\frac{\ln t}{t^2+1}dt&&\displaystyle \mathbf 2.\ \int_1^{+\infty}\frac{\sqrt{\ln x}}{(x-1)\sqrt x}dx\\ \displaystyle \mathbf 3. \int_1^{+\infty} e^{-\sqrt{\ln t}}dt \end{array}$$
Indication
Corrigé
Exercice 4 - Convergence d'intégrales impropres - avec paramètres [Signaler une erreur] [Ajouter à ma feuille d'exos]
Enoncé
Discuter, suivant la valeur du paramètre $\alpha\in\mathbb R$, la convergence des intégrales impropres suivantes : $$\begin{array}{lll} \displaystyle \mathbf 1.\ \int_0^{+\infty}\frac{dt}{t^\alpha}&&\displaystyle \mathbf2.\ \int_0^{+\infty}\frac{e^{-t}-1}{t^\alpha}dt\\ \displaystyle \mathbf 3.\ \int_0^{+\infty}\frac{t-\sin t}{t^\alpha}dt&& \displaystyle \mathbf 4.\ \int_0^{+\infty}\frac{\arctan t}{t^\alpha}dt \end{array}$$
Indication
Corrigé
Exercice 5 - Convergence et logarithme [Signaler une erreur] [Ajouter à ma feuille d'exos]
Enoncé
  1. Démontrer la convergence de l'intégrale $\int_0^1 \frac{\ln x}{x^{3/4}}dx$. On pourra comparer avec $\frac 1{x^\alpha}$ pour $\alpha$ bien choisi.
  2. Donner un équivalent simple au voisinage de $0$ de $\ln\left(x+\sqrt x\right)-\ln(x)$. En déduire la convergence de $\int_0^1\frac{\ln\left(x+\sqrt x\right)-\ln(x)}{x^{3/4}}dx$.
  3. Donner un équivalent simple au voisinage de $+\infty$ de $\ln\left(x+\sqrt x\right)-\ln(x)$. En déduire la nature de $\int_1^{+\infty}\frac{\ln\left(x+\sqrt x\right)-\ln(x)}{x^{3/4}}dx$.
Indication
Corrigé
Enoncé
  1. La fonction $\displaystyle x\mapsto \frac{e^{-x}}{\sqrt{x^2-4}}$ est-elle intégrable sur $]2,+\infty[$?
  2. Soit $a$ un réel positif. La fonction $\displaystyle x\mapsto\frac{\ln(x)}{\sqrt{1+x^{2a}}}$ est-elle intégrable sur $]0,+\infty[$?
Corrigé
Exercice 7 - Intégrales de Bertrand [Signaler une erreur] [Ajouter à ma feuille d'exos]
Enoncé
Pour $\alpha,\beta\in\mathbb R$, on souhaite déterminer la nature de $$\int_e^{+\infty}\frac{dx}{x^\alpha(\ln x)^\beta}.$$
  1. On suppose $\alpha>1$. En comparant avec une intégrale de Riemann, démontrer que l'intégrale étudiée est convergente.
  2. On suppose $\alpha=1$. Calculer, pour $X>e$, $\int_e^X\frac{dx}{x(\ln x)^\beta}$. En déduire les valeurs de $\beta$ pour lesquelles l'intégrale converge.
  3. On suppose $\alpha<1$. En comparant à $1/t$, démontrer que l'intégrale étudiée diverge.
Indication
Corrigé
Exercice 8 - Convergence d'intégrales impropres à paramètres - 2 [Signaler une erreur] [Ajouter à ma feuille d'exos]
Enoncé
Discuter, suivant la valeur de $\alpha\in\mathbb R$, la convergence des intégrales suivantes : $$\begin{array}{lll} \displaystyle \mathbf 1.\ \int_0^{+\infty}\frac{t\ln t}{(1+t^2)^\alpha}dt&& \displaystyle \mathbf 2.\ \int_0^{+\infty}x^\alpha\ln\left(x+e^{\alpha x}\right)dx\ \end{array}$$
Corrigé
Exercice 9 - Convergence d'intégrales impropres avec développements limités [Signaler une erreur] [Ajouter à ma feuille d'exos]
Enoncé
Les intégrales impropres suivantes sont-elles convergentes? $$\begin{array}{lll} \displaystyle \mathbf 1.\ \int_0^1 \frac{dt}{1-\sqrt t}&&\displaystyle \mathbf 2.\ \int_0^{+\infty}\left(1+t\ln\left(\frac{t}{t+1}\right)\right)dt\\ \displaystyle \mathbf 3.\ \int_2^{+\infty}\left(\sqrt{x^4+x^2+1}-x\sqrt[3]{x^3+ax}\right)dx,\ a\in\mathbb R.&&\displaystyle \mathbf 4.\ \int_0^{+\infty}e^{-t}\left(\frac1{1-e^{-t}}-\frac 1t\right)dt. \end{array}$$
Indication
Corrigé
Enoncé
  1. Soit $f:[0,+\infty[\to\mathbb R$ une fonction continue. On suppose que $\int_0^{+\infty}f(t)dt$ converge, et soit $(x_n)$ et $(y_n)$ deux suites tendant vers $+\infty$. Démontrer que $\int_{x_n}^{y_n}f(t)dt$ tend vers 0.
  2. En déduire que l'intégrale $\int_0^{+\infty}e^{-t\sin t}dt$ diverge.
Indication
Corrigé
Exercice 11 - Convergence et convergence absolue [Signaler une erreur] [Ajouter à ma feuille d'exos]
Enoncé
  1. Montrer que les intégrales impropres $\int_1^{+\infty}\frac{\sin t}{t}dt$ et $\int_1^{+\infty}\frac{\cos t}tdt$ sont convergentes.
    On souhaite prouver que la fonction $\frac{\sin t}{t}$ n'est pas intégrable, c'est-à-dire que $\int_1^{+\infty}\left|\frac{\sin t}t\right|dt$ diverge.
  2. Méthode 1. Prouver que, pour tout $t\in\mathbb R$, $|\sin t|\geq \frac{1-\cos 2t}{2}$. En déduire le résultat.
  3. Méthode 2. Prouver que, pour tout $k\in\mathbb N$, $$\int_{k\pi}^{(k+1)\pi}\frac{|\sin t|}{t}dt\geq\frac{1}{(k+1)\pi}\int_0^{\pi}|\sin t|dt.$$ Retrouver alors le résultat.
Indication
Corrigé
Exercice 12 - Méthode par éclatement [Signaler une erreur] [Ajouter à ma feuille d'exos]
Enoncé
Étudier la convergence des intégrales suivantes : $$\begin{array}{lll} \displaystyle \mathbf 1. \int_4^{+\infty}\frac{\sin x}{\sqrt x+\sin x }dx&&\displaystyle \mathbf 2.\ \int_1^{+\infty}\ln\left(1+\frac{\sin x}{x^\alpha}\right)dx,\ \alpha>0 \end{array}$$
Indication
Corrigé
Exercice 13 - Transformée de Laplace [Signaler une erreur] [Ajouter à ma feuille d'exos]
Enoncé
Soit $f:[0,+\infty[\to\mathbb R$ une fonction continue et $s_0\in\mathbb R$ tels que $\int_0^{+\infty}f(t)e^{-s_0t}dt$ converge.
  1. Soit $F$ une primitive de $t\mapsto f(t)e^{-s_0t}$ sur $[0,+\infty[$. Démontrer que $F$ est bornée sur $[0,+\infty[$.
  2. En déduire que, pour tout $s>s_0$, $\int_0^{+\infty}f(t)e^{-st}dt$ converge.
  3. Sur le même modèle, démontrer que si $g:[1,+\infty[\to\mathbb R$ est une fonction continue telle que $\int_1^{+\infty}g(t)dt$ converge, alors $\int_1^{+\infty}\frac{g(t)}tdt$ converge.
Indication
Corrigé
Exercice 14 - Avec le critère des séries alternées [Signaler une erreur] [Ajouter à ma feuille d'exos]
Enoncé
Soit $f:[0,+\infty[\to[0,+\infty[$ une fonction continue décroissante, de limite nulle en $+\infty$. On pose $u_n=\int_{n\pi}^{(n+1)\pi}f(t)\sin(t)dt$.
  1. Montrer que la série de terme général $u_n$ est convergente.
  2. En déduire que l'intégrale $\int_0^{+\infty}f(t)\sin(t)dt$ est convergente. Quel est son signe?
  3. On suppose $f(x)\geq 1/x$ pour $x\geq x_0$. Prouver que $\int_0^{+\infty}f(t)\sin(t)dt$ n'est pas absolument convergente.
Indication
Corrigé
Exercice 15 - Intégrale impropre et fonction $1-$périodique. [Signaler une erreur] [Ajouter à ma feuille d'exos]
Enoncé
Soit $f:\mathbb R\to\mathbb R$ continue et $1$-périodique. Déterminer l'ensemble des $a\in\mathbb R$ tels que $\int_1^{+\infty}\frac{a-f(t)}{t}dt$ converge.
Indication
Corrigé
Exercice 16 - Utilisation d'une série [Signaler une erreur] [Ajouter à ma feuille d'exos]
Enoncé
Soit $f$ la fonction définie sur $\mathbb R$ par $$f(x)=\frac{1}{1+x^4\sin^2 x}.$$ Démontrer que $f$ est intégrable sur $\mathbb R$.
Indication
Corrigé
Calcul
Exercice 17 - Logarithme à la puissance $n$ [Signaler une erreur] [Ajouter à ma feuille d'exos]
Enoncé
Après en avoir justifié l'existence, calculer par récurrence la valeur de $I_n=\int_0^1 (\ln x)^ndx.$
Indication
Corrigé
Exercice 18 - Changements de variables [Signaler une erreur] [Ajouter à ma feuille d'exos]
Enoncé
  1. Montrer que $\int_0^{+\infty}\frac{\ln t}{1+t^2}dt$ converge, puis, avec le changement de variables $u=1/t$, que $\int_0^{+\infty}\frac{\ln t}{1+t^2}dt=0$.
  2. Soit $a>0$. Calculer $\int_0^{+\infty}\frac{\ln t}{a^2+t^2}dt$.
Indication
Corrigé
Exercice 19 - Changement de variable [Signaler une erreur] [Ajouter à ma feuille d'exos]
Enoncé
Soit $f$ une fonction continue bornée sur $[0,+\infty[$.
  1. Démontrer que les intégrales $\int_0^{+\infty}\frac{f(x)}{1+x^2}dx$ et $\int_0^{+\infty}\frac{f(1/x)}{1+x^2}dx$ sont convergentes.
  2. Démontrer qu'elles sont égales.
  3. Application : pour $n\geq 0$, calculer $\int_0^{+\infty}\frac{dx}{(1+x^2)(1+x^n)}$ et $\int_0^{+\infty}\frac{x^n}{(1+x^2)(1+x^n)}dx.$
Indication
Corrigé
Exercice 20 - Différence d'exponentielles [Signaler une erreur] [Ajouter à ma feuille d'exos]
Enoncé
Soient $0<a<b$.
  1. Justifier la convergence de $\int_0^{+\infty}\frac{e^{-at}-e^{-bt}}tdt$.
  2. Soient $0<x<y$. Démontrer que $$\int_x^y \frac{e^{-at}-e^{-bt}}tdt=\int_{ax}^{bx}\frac{e^{-t}}tdt-\int_{ay}^{by}\frac{e^{-t}}tdt.$$
  3. Démontrer que, pour tout réel $z>0$, $$e^{-bz}\ln\frac ba\leq\int_{az}^{bz}\frac{e^{-t}}tdt\leq e^{-az}\ln\frac ba.$$
  4. En déduire que $$\int_0^{+\infty}\frac{e^{-at}-e^{-bt}}tdt=\ln\frac ba.$$
Indication
Corrigé
Exercice 21 - Une intégrale comme somme d'une série [Signaler une erreur] [Ajouter à ma feuille d'exos]
Enoncé
Le but de l'exercice est de prouver la relation suivante : $$\int_0^1\frac{\ln t}{t^2-1}dt=\lim_{n\to+\infty}\sum_{k=0}^n\frac{1}{(2k+1)^2}.$$
  1. Prouver la convergence de l'intégrale.
  2. Montrer que, pour tout entier $k\geq 0$, l'intégrale $I_k=\int_0^1 t^k\ln tdt$ converge, puis calculer $I_k$.
  3. Montrer que, pour tout entier $n\geq 1$, $\sum_{k=0}^n\frac{1}{(2k+1)^2}=\int_0^1 \frac{\ln t}{t^2-1}dt-\int_0^1 \frac{t^{2n+2}\ln t}{t^2-1}dt.$
  4. Démontrer que la fonction $t\mapsto \frac{t^2\ln t}{t^2-1}$ se prolonge par continuité en 0 et en 1. En déduire qu'il existe une constante $M>0$, qu'on ne cherchera pas à calculer, telle que, pour tout $t\in]0,1[$, $\left|\frac{t^2\ln t}{t^2-1}\right|\leq M$.
  5. En déduire que $\lim_{n\to+\infty}\int_0^1\frac{t^{2n+2}\ln t}{t^2-1}dt=0$, puis la relation demandée.
Indication
Corrigé
Exercice 22 - Différence d'arctangente [Signaler une erreur] [Ajouter à ma feuille d'exos]
Enoncé
  1. Démontrer la convergence de $\int_0^{+\infty}\big(\arctan(x+1)-\arctan(x)\big)dx$.
  2. Démontrer que $\lim_{X\to +\infty}\int_X^{X+1}\arctan(x)dx=\frac\pi 2$.
  3. Calculer $\int_0^1 \arctan(x)dx$.
  4. Calculer $\int_0^{+\infty}\big(\arctan(x+1)-\arctan(x)\big)dx$
Indication
Corrigé
Enoncé
Justifier la convergence et calculer la valeur des intégrales suivantes : $$\begin{array}{lll} \displaystyle \mathbf 1.\ \int_0^{1}\frac{\ln t}{\sqrt{1-t}}dt&&\displaystyle \mathbf 2.\ \int_0^{+\infty}te^{-\sqrt t}dt\\ \displaystyle\mathbf 3.\int_0^{+\infty}\sin(t)e^{-at}dt,\ a>0. \end{array}$$
Indication
Corrigé
Exercice 24 - Intégrale de Dirichlet [Signaler une erreur] [Ajouter à ma feuille d'exos]
Enoncé
Le but de cet exercice est de calculer la valeur de $I=\int_0^{+\infty}\frac{\sin t}tdt$. Pour chaque entier $n$, on note $$I_n=\int_0^{\pi/2}\frac{\sin \big((2n+1)t\big)}{\sin t}dt\textrm{ et }J_n=\int_0^{\pi/2}\frac{\sin \big((2n+1)t\big)}{t}dt.$$
  1. Justifier que, pour tout $n\geq 0$, $I_n$ et $J_n$ sont bien définis.
  2. Montrer que, pour tout $n\geq 1$, $I_n-I_{n-1}=0$. En déduire la valeur de $I_n$.
  3. Soit $\phi:[0,\pi/2]\to\mathbb R$ de classe $C^1$. Montrer, à l'aide d'une intégration par parties, que $\int_0^{\pi/2}\phi(t)\sin\big((2n+1)t\big)dt$ tend vers 0.
  4. Démontrer que la fonction $t\mapsto \frac 1t-\frac 1{\sin t}$ se prolonge en une fonction de classe $C^1$ sur $[0,\pi/2]$.
  5. En déduire que $J_n-I_n\to 0$.
  6. Démontrer, en utilisant un changement de variables, que $J_n\to I$.
  7. En déduire la valeur de $I$.
Indication
Corrigé
Exercice 25 - Application à la positivité de polynômes [Signaler une erreur] [Ajouter à ma feuille d'exos]
Enoncé
Soit $P\in\mathbb R[X]$ tel que, pour tout $x\in\mathbb R,$ $P(x)\geq 0$. On note $n=\textrm{deg}(P)$ et $Q=\sum_{k=0}^n P^{(k)}$.
  1. Démontrer que, pour tout $x\in\mathbb R,$ $$Q(x)=e^{x}\int_x^{+\infty}e^{-t}P(t)dt.$$
  2. En déduire que $Q\geq 0$.
Indication
Corrigé
Exercice 26 - $\big(\arctan(2x)-\arctan(x)\big)/x$ [Signaler une erreur] [Ajouter à ma feuille d'exos]
Enoncé
Calculer $\displaystyle \int_0^{+\infty}\frac{\arctan(2x)-\arctan(x)}xdx$.
Indication
Corrigé
Enoncé
  1. Déterminer $\displaystyle \lim_{x\to 0^+}\int_x^{3x}\frac{\sin(t)}{t^2}dt.$
  2. Justifier la convergence de l'intégrale $\displaystyle \int_0^{+\infty}\frac{\sin^3(t)}{t^2}dt.$
  3. En linéarisant $\sin^3(t)$, calculer cette intégrale.
Indication
Corrigé
Intégration des relations de comparaison
Exercice 28 - Intégration des relations de comparaison [Signaler une erreur] [Ajouter à ma feuille d'exos]
Enoncé
Soient $f$ et $g$ deux fonctions continues par morceaux sur $[a,b[$, à valeurs dans $\mathbb R_+$.
  1. On suppose dans cette question que $g=_bo(f)$.
    1. On suppose que $\int_a^b f(t)dt$ converge. Montrer que $$\int_x^b g(t)dt=_bo\left(\int_x^b f(t)dt\right).$$
    2. On suppose que $\int_a^b g(t)dt$ diverge. Montrer que $$\int_a^x g(t)dt=_bo\left(\int_a^x f(t)dt\right).$$
  2. On suppose désormais que $g\sim_b f$. Déduire de la question précédente que,
    1. si $\int_a^b f(t)dt$ converge, alors $$\int_x^b g(t)dt\sim_b \int_x^b f(t)dt ;$$
    2. si $\int_a^b f(t)dt$ diverge, alors $$\int_a^x g(t)dt\sim_b \int_a^x f(t)dt;$$
  3. Donner un équivalent de $\int_1^{x}\frac{\arctan t}{t}dt$ lorsque $x$ tend vers $+\infty$.
Indication
Corrigé
Enoncé
Donner un équivalent de $\int_1^{x}\frac{\arctan t}{t}dt$ lorsque $x$ tend vers $+\infty$.
Indication
Corrigé
Exercice 30 - Équivalent de la queue de la gaussienne [Signaler une erreur] [Ajouter à ma feuille d'exos]
Enoncé
  1. Justifier la convergence de $\int_0^{+\infty}e^{-t^2}dt$.
  2. Démontrer que, pour tout $x>0$, on a $$\int_x^{+\infty}e^{-t^2}dt=\frac{e^{-x^2}}{2x}-\int_x^{+\infty}\frac{e^{-t^2}}{2t^2}dt.$$
  3. En déduire un équivalent simple de $\int_x^{+\infty}e^{-t^2}dt$ lorsque $x$ tend vers $+\infty$.
Indication
Corrigé
Exercice 31 - Équivalent du reste de l'intégrale de $e^{-t}/t$ [Signaler une erreur] [Ajouter à ma feuille d'exos]
Enoncé
Déterminer un équivalent simple en $+\infty$ de $\int_x^{+\infty}\frac{e^{-t}}tdt$.
Indication
Corrigé
Exercice 32 - Développement asymptotique d'une intégrale [Signaler une erreur] [Ajouter à ma feuille d'exos]
Enoncé
Donner un développement asymptotique à trois termes au voisinage de $+\infty$ de $\int_1^x \frac{e^t}tdt$.
Indication
Corrigé
Exercice 33 - Développement asymptotique du logarithme intégral [Signaler une erreur] [Ajouter à ma feuille d'exos]
Enoncé
On considère le logarithme intégral qui est la fonction définie par $$\forall x\geq 2,\ li(x)=\int_2^x \frac{dt}{\ln t}.$$ Pour tout $n\geq 1$, donner un développement asymptotique de $li(x)$ à $n$ termes lorsque $x\to+\infty$.
Indication
Corrigé
Autres estimations de restes ou d'intégrales partielles
Enoncé
Soit $f:[0,+\infty[\to\mathbb R$ une fonction continue, et $b>a>0$ deux réels.
  1. On suppose que $f(0)=0$. Démontrer que $$\lim_{x\to 0^+}\int_{ax}^{bx}\frac{f(t)}tdt=0.$$
  2. Déterminer $\lim_{x\to 0^+}\int_{ax}^{bx}\frac{f(t)}tdt$ si on ne suppose plus que $f(0)=0$.
Indication
Corrigé
Enoncé
Déterminer la limite, lorsque $x\to 0^+$, de $\int_x^{2x}\frac{\sin t}{t^2}dt$.
Indication
Corrigé
Enoncé
Soit $0<a<b$. Déterminer un équivalent, lorsque $x\to+\infty$ de $\displaystyle \int_{ax}^{bx}\frac{\ln(1+t)}{t}dt.$
Indication
Corrigé
Application à l'étude de fonctions définies par des intégrales
Enoncé
  1. Pour quelles valeurs de $a\in\mathbb R$ l'intégrale impropre $\int_0^{+\infty}e^{-ax}dx$ est-elle convergente?
  2. Pour quelles valeurs de $a\in\mathbb R$ l'intégrale impropre $\int_0^{+\infty}e^{-ax}\arctan xdx$ est-elle convergente? On note $\mathcal D$ cet ensemble de valeurs et pour $a\in\mathcal D$, on note $I(a)$ la valeur de l'intégrale impropre.
  3. Soit $a\in\mathcal D$. Démontrer que $\displaystyle I(a)=\frac1{a^2}-\frac{2}{a^2}\int_0^{+\infty}\frac{xe^{-ax}}{(1+x^2)^2}dx$.
  4. Démontrer que la fonction $\displaystyle x\mapsto \frac{x}{(1+x^2)^2}$ est bornée sur $\mathbb R_+$.
  5. En déduire que $\displaystyle \lim_{a\to+\infty}\int_0^{+\infty}\frac{xe^{-ax}}{(1+x^2)^2}dx=0$.
  6. Déterminer un équivalent simple de $I(a)$ lorsque $a$ tend vers $+\infty$.
Indication
Corrigé
Exercice 38 - Une intégrale dépendant d'un paramètre [Signaler une erreur] [Ajouter à ma feuille d'exos]
Enoncé
Soit $x\in\mathbb R$. On définit, lorsque l'intégrale est convergente, la fonction $$\phi(x)=\int_1^{+\infty}\frac{dt}{1+t^x}.$$
  1. Déterminer le domaine de définition de $\phi$.
  2. Démontrer que $\phi$ est décroissante sur son domaine de définition.
  3. Déterminer la limite de $\phi$ en $+\infty$.
Indication
Corrigé
Enoncé
On note, pour tout $n\in\mathbb N^*$, sous réserve d'existence, $\displaystyle I_n=\int_0^{+\infty}\frac{e^{-x}}{x+n}dx.$
  1. Justifier l'existence de $I_n$ pour tout $n\in\mathbb N^*$.
  2. Établir que $(I_n)$ converge vers $0$.
  3. Démontrer que $I_n\sim_{+\infty}\frac 1n.$
Indication
Corrigé
Exercice 40 - Lemme de Riemann-Lebesgue [Signaler une erreur] [Ajouter à ma feuille d'exos]
Enoncé
Soit $f:[0,+\infty[\to\mathbb R$ une fonction de classe $\mathcal C^1$ intégrable.
  1. Démontrer que, pour tout $A>0$, $\int_0^A f(t)\cos(xt)dt$ tend vers 0 lorsque $x$ tend vers $+\infty$.
  2. En déduire que $\int_0^{+\infty} f(t)\cos(xt)dt$ tend vers 0 lorsque $x$ tend vers $+\infty$.
Indication
Corrigé
Exercices théoriques
Exercice 41 - Critère de Cauchy (sens facile) [Signaler une erreur] [Ajouter à ma feuille d'exos]
Enoncé
Soit $f$ une fonction continue par morceaux sur $[0,+\infty[$. On suppose que $f$ est intégrable sur $[0,+\infty[$. Démontrer que $\int_x^{x+1}f(t)dt\xrightarrow{x\to+\infty}0$.
Corrigé
Exercice 42 - Intégrabilité par encadrement [Signaler une erreur] [Ajouter à ma feuille d'exos]
Enoncé
Soit $I$ un intervalle et $f,g,h:I\to\mathbb R$ continues par morceaux. On suppose que $f$ et $h$ sont intégrables sur $I$ et que $f\leq g\leq h.$ Démontrer que $g$ est intégrable sur $I$.
Indication
Corrigé
Enoncé
Soit $f:[0,+\infty[\to\mathbb R$ une fonction de classe $\mathcal C^1$ telle que $f$ et $f'$ soient intégrables sur $[0,+\infty[$. Démontrer que $f$ tend vers $0$ en $+\infty$.
Indication
Corrigé
Exercice 44 - Comportement en l'infini [Signaler une erreur] [Ajouter à ma feuille d'exos]
Enoncé
Soit $f:[0,+\infty[\to\mathbb R$ continue par morceaux et intégrable.
  1. Démontrer que, pour tout $A>0$ et tout $\veps>0$, il existe $x\geq A$ tel que $|xf(x)|\leq \veps$.
  2. En déduire l'existence d'une suite $(x_n)$ tendant vers $+\infty$ telle que $\big(x_nf(x_n)\big)$ tend vers 0.
Indication
Corrigé
Exercice 45 - Fonction décroissante [Signaler une erreur] [Ajouter à ma feuille d'exos]
Enoncé
Soit $f:[0,+\infty[\to\mathbb R$ une fonction continue décroissante telle que $\int_0^{+\infty} f(t)dt$ converge.
  1. Démontrer que $f\geq 0$.
  2. Démontrer que $f$ tend vers 0 en $+\infty$.
  3. Justifier que $\int_{x/2}^x f(t)dt$ tend vers 0 lorsque $x$ tend vers $+\infty$.
  4. En déduire que $xf(x)$ tend vers 0 lorsque $x$ tend vers $+\infty$.
Indication
Corrigé
Exercice 46 - Fonction intégrable et limites en l'infini [Signaler une erreur] [Ajouter à ma feuille d'exos]
Enoncé
Soit $a$ un réel et $f$ une application continue de $[a,+\infty[$ dans $\mathbb R$, intégrable sur $[a,+\infty[$.
  1. Montrer que si $f$ admet une limite en $+\infty$, cette limite est nécessairement nulle.
  2. Montrer que si $f$ est uniformément continue, alors elle tend vers 0 en $+\infty$.
  3. Le résultat subsiste-t-il si on suppose simplement $f$ continue?
Indication
Corrigé
Enoncé
Soit $f:[a,b[\to\mathbb R_+$ continue et croissante. On note $S_n=\sum_{k=0}^{n-1}\frac{b-a}nf\left(a+\frac{k(b-a)}n\right)$.
  1. On suppose que $\int_a^b f(t)dt$ converge. Montrer que la suite $(S_n)$ converge vers $\int_a^b f(t)dt$.
  2. On suppose que $\int_a^b f(t)dt$ diverge. Montrer que la suite $(S_n)$ tend vers $+\infty$.
Indication
Corrigé
Enoncé
Soit $f$ une fonction continue de carré intégrable de $[0,+\infty[$ dans $\mathbb R$.
  1. Prouver que, pour tous $0\leq a\leq b$, on a $$\left|\int_a^b f(t)dt\right|\leq \sqrt{b-a}\left(\int_a^b f^2(t)dt\right)^{1/2}.$$
  2. En déduire que $$\lim_{x\to+\infty}\frac1{\sqrt x}\int_0^x f(t)dt=0.$$
Indication
Corrigé
Enoncé
Soit $f:[0,+\infty[\to]0,+\infty[$ de classe $C^1$ telle qu'il existe $a<0$ satisfaisant $\lim_{x\to+\infty}\frac{f'(x)}{f(x)}= a$. Montrer que $f$ et $f'$ sont intégrables sur $[0,+\infty[$.
Indication
Corrigé
Enoncé
Soit $f\in\mathcal C^2(\mathbb R,\mathbb R)$ telle que $f$ et $f''$ sont de carré intégrable.
  1. Démontrer que $f'$ est de carré intégrable.
  2. Démontrer que $\left(\int_{\mathbb R}f'^2\right)^2\leq \left(\int_{\mathbb R}f^2\right)\left(\int_{\mathbb R}f''^2\right).$
Indication
Corrigé
Petits problèmes
Exercice 51 - Exercice de synthèse sur l'intégration [Signaler une erreur] [Ajouter à ma feuille d'exos]
Enoncé
Pour $x>0$, on note $\varphi(x)=\exp\left(\frac 1{\sqrt x}\right)$ et $f(x)=\int_{x}^{2x}\varphi(t)dt$.
  1. Justifier que $f$ est bien définie sur $]0,+\infty[$.
  2. Exprimer $f$ en fonction d'une primitive $\phi$ de $\varphi$. En déduire que $f$ est de classe $C^1$ sur $]0,+\infty[$ et calculer sa dérivée.
  3. Montrer que, pour tout $x>0$, $f'(x)$ est du signe de $u(x)=2-\exp\left(\frac\alpha{\sqrt x}\right)$ avec $\alpha=1-\frac1{\sqrt 2}$. En déduire les variations de $f$ sur $]0,+\infty[$.
  4. Démontrer que, pour tout $x>0$, $$x \exp\left(\frac 1{\sqrt{2x}}\right)\leq f(x)\leq x\exp\left(\frac 1{\sqrt x}\right).$$
  5. En déduire les limites de $f$ en $0$ et en $+\infty$, puis un équivalent de $f(x)$ en $+\infty$.
  6. Quelle est la nature de l'intégrale généralisée $\int_1^{+\infty}f(x)dx$?
  7. Quelle est la nature de l'intégrale généralisée $\int_0^1 f(x)dx$ (on pourra faire le changement de variables $u=1/\sqrt x$)?
Corrigé
Enoncé
Dans toute la suite, $f:\mathbb R\to\mathbb R$ désigne une fonction continue et $a$ un réel strictement positif. Pour tout $\lambda$ réel, on note $I(\lambda)=\int_a^{+\infty}\frac{\lambda-f(t)}{t}dt$ lorsque cette intégrale impropre existe.
  1. On suppose qu'il existe deux réels $\lambda$ et $\mu$ pour lesquels $I(\lambda)$ et $I(\mu)$ convergent. Démontrer que $\lambda=\mu$.
  2. Pour tout $x$ réel, on pose $H_\lambda(x)=\int_a^x \big(\lambda-f(t)\big)dt$.
    1. Justifier que $H_\lambda$ est de classe $\mathcal C^1$ sur $\mathbb R$ et préciser $H_\lambda'(x)$.
    2. Démontrer que si $H_\lambda$ est bornée sur $\mathbb R$, alors $I(\lambda)$ existe et $I(\lambda)=\int_a^{+\infty}\frac{H_{\lambda}(t)}{t^2}dt$.
  3. Désormais, on suppose en outre que $f$ est $T$-périodique, $T>0$.
    1. Démontrer que la fonction $\varphi$ qui à tout $x$ réel associe $\varphi(x)=\int_x^{x+T}f(t)dt$ est constante. Montrer alors que l'on a, pour tout réel $x$, $$H_\lambda(x+T)-H_\lambda(x)=\lambda T-\int_0^T f(t)dt.$$
    2. Démontrer qu'il existe une unique valeur $\lambda_0$ du réel $\lambda$ pour laquelle la suite $(H_\lambda(a+nT))_{n\in\mathbb N}$ est bornée.
    3. Prouver que, dans ce cas, la fonction $H_\lambda$ est périodique et bornée dans $\mathbb R$.
    4. Déterminer alors toutes les valeurs du réel $\lambda$ pour lesquelles $I(\lambda)$ converge.
    5. Dans le cas où $\lambda_0\neq 0$, déterminer un équivalent de $\int_a^x \frac{f(t)}{t}dt$ lorsque $x$ tend vers l'infini.
Indication
Corrigé
Enoncé
  1. Montrer que pour tout $x>0$, l'intégrale $\displaystyle\int_x^{+\infty}\frac{e^{-t}}{t}\,dt$ est convergente.
    On pose $F(x)=\displaystyle\int_x^{+\infty}\frac{e^{-t}}{t}\,dt$ si $x>0$.
  2. Montrer que $F$ est de classe $C^1$ sur $]0,+\infty[$ et calculer $F'$.
  3. Calculer $\displaystyle\lim_{x\to 0^+}F(x)$ et $\displaystyle\lim_{x\to +\infty}F(x)$.
  4. On cherche un équivalent de $F(x)$ lorsque $x\to 0^+$.
    1. Démontrer que la fonction $t\mapsto \frac{e^{-t}-1}{t}$ se prolonge par continuité en $0$.
    2. Démontrer qu'il existe une constante $C>0$ telle que, pour tout $x\in]0,1]$, $$\left|\int_x^1 \frac{e^{-t}-1}{t}dt\right|\leq C.$$
    3. En déduire que $F(x)\sim -\ln x$ lorsque $x\to 0^+$.
  5. On cherche un équivalent de $F(x)$ lorsque $x\to +\infty$.
    1. Montrer que pour tout $x>0$, l'int\'egrale $\displaystyle\int_x^{+\infty}\frac{e^{-t}}{t^2}\,dt$ est convergente.
    2. Montrer que pour tout $x>0$, $\displaystyle\int_x^{+\infty}\frac{e^{-t}}{t^2}\,dt \le \frac1xF(x)$.
    3. A l'aide d'une intégration par parties, en déduire que $F(x)\sim \frac{e^{-x}}{x}$ lorsque $x\to +\infty$.
Corrigé
Enoncé
L'objectif de ce problème est l'étude de la fonction dilogarithme définie par : $$Li(x)=\int_1^x \frac{\ln(t)}{1-t}dt.$$
  1. Questions préliminaires : on pose, pour $t>0$ et $t\neq 1$ et pour $x> 0$, $$f(t)=\frac{\ln(t)}{1-t},\ g(x)=\frac{x}{e^x-1}.$$
    1. Démontrer que $f$ se prolonge par continuité en 1 et que $g$ se prolonge par continuité en $0$.
    2. Démontrer que, pour tout $x\geq 0$, $0\leq g(x)\leq 1$.
  2. Démontrer que le domaine de définition de la fonction dilogarithme est $[0,+\infty[$.
  3. Justifier soigneusement que la fonction dilogarithme est dérivable sur l'intervalle $]0,+\infty[$ et donner l'expression de sa dérivée.
  4. Pourquoi $Li$ est-elle continue en $0$?
  5. Dans cette question, on se propose de calculer $Li(0)$.
    1. Démontrer que $Li(0)=\int_0^{+\infty}\frac{x}{e^x-1}dx$.
    2. Justifier que, pour tout $k\geq 1$, l'intégrale $\int_0^{+\infty}xe^{-kx}dx$ est convergente, et calculer sa valeur.
    3. Soit $(S_n)$ la suite définie pour tout $n\geq 1$ par $$S_n=\sum_{k=1}^n \int_0^{+\infty}xe^{-kx}dx.$$ Démontrer que $$Li(0)-S_n=\int_0^{+\infty}\frac{xe^{-nx}}{e^x-1}dx.$$
    4. En déduire que $$0\leq Li(0)-S_n\leq\frac 1n.$$
    5. En admettant que $$\sum_{k\geq 1}\frac 1{k^2}=\frac{\pi^2}6,$$ démontrer que $$Li(0)=\frac{\pi^2}6.$$
  6. Quelle est la nature de $\int_1^{+\infty}\frac{\ln t}{1-t}dt$? En déduire la limite de $Li(x)$ quand $x$ tend vers $+\infty$.
  7. Donner la représentation graphique de la fonction $Li$.
  8. On pose, pour tout $x\in ]0,1[$, $$u(x)=Li(x)+Li(1-x),$$ $$v(x)=-\ln(1-x)\ln(x).$$ Démontrer qu'il existe une constante $C\in\mathbb R$ telle que, pour tout $x\in ]0,1[$, $$u(x)=v(x)+C.$$
  9. Déterminer $C$.
Indication
Corrigé